タグ付けされた質問 「lattice」

1
SATに関連するトポロジ空間:コンパクトですか?
充足可能性問題は、もちろん、理論的なCSでの根本的な問題です。私は無限に多くの変数を持つ問題の1つのバージョンで遊んでいました。\newcommand{\sat}{\mathrm{sat}} \newcommand{\unsat}{\mathrm{unsat}} 基本セットアップ。ましょ空でないとすることの可能性が無限集合変数。リテラルは、変数x \ in Xまたはその否定\ neg xのいずれかです。節cは、有限数のリテラルの分離です。最後に、式Fを一連のClauseとして定義します。バツバツX¬ X CX ∈ Xバツ∈バツx \in X¬ X¬バツ\neg xcccFFF Xの割り当てバツバツXは、関数σ:X→ { 0 、1 }σ:バツ→{0、1}\sigma : X \to \{0,1\}です。割り当てσσ\sigmaが句を満たすときの条件を明示的に定義しません。これは少し面倒で、標準のSATと同じです。最後に、すべての構成句を満たす場合、割り当ては式を満たします。してみましょうs a t( F)sat(F)\sat(F)の割り当てを満たすの集合FFF、としましょうu n s a t( F)あなたはnsat(F)\unsat(F)の補完するs a t( F)sat(F)\sat(F)。 トポロジー空間。 私たちの目標は、Xのすべての割り当ての空間を与えることです。これをトポロジ構造で\ SigmaバツバツXと呼びます。閉集合の形式は\ sat(F)で、Fは式です。これが実際にトポロジであることを確認できます。ΣΣ\Sigmas a t( F)sat(F)\sat(F)FFF 句を含まない空の式∅∅\emptysetは、すべての割り当てで満たされます。そうΣΣ\Sigma閉じられています。 式{ X 、¬ X }{バツ、¬バツ}\{ x, …

4
理論CSにおけるポーズ/格子上の計量構造の応用
この用語はオーバーロードされているため、最初に簡単な定義から始めます。ポーズは、部分順序付与されたセットです。二つの要素所与、我々は定義することができ上部に結合し、それらの少なくともとして(参加)を、と同様に定義する下限最大として(結合)(出会う)を。≤ 、B ∈ X X ∨ Y X X ∧ YバツバツX≤≤\le、B ∈ Xa、b∈バツa,b \in XX ∨ Yバツ∨yx \vee yバツバツXX ∧ Yバツ∧yx \wedge y ラティスは、任意の2つの要素が一意のミートと一意の結合を持つポーズです。 格子(この形式)は、(簡単に)準モジュラリティ(サブセットラティスを含む)およびクラスタリング(パーティションラティス)の理論CS、およびドメイン理論(あまりよく理解していません)および静的に表示されます分析。 しかし、格子上のメトリック構造を使用するアプリケーションに興味があります。単純な例は、任意の反単調サブモジュラー関数(反単調は、場合が計量 X ≤ Y 、F (X )≤ F (Y )D (X 、Y )= 2 、F (X ∧ Y )- 、F (X )- F (Y )f:X→ …

3
2つのパーティション間の距離を編集する
2つのパーティションがあり[1…n][1…n][1 \ldots n]、それらの間の編集距離を探しています。 これによって、パーティションAからパーティションBに移動するために必要な、ノードの異なるグループへの単一の遷移の最小数を見つけたいと思います。 たとえば、から{0 1} {2 3} {4}への距離は{0} {1} {2 3 4}2 検索した後、私はこの論文に出くわしましたが、a)彼らが遠くにいるグループ(私は気にしない)の順序を考慮に入れているかどうかはわかりませんb)それがどのように機能するかはわかりませんc)参照はありません。 助けていただければ幸いです

2
編集(タラB著):私は自分の論文のためにそれを自分自身で証明しなければならなかったので、この証拠への参照にまだ興味があります。 この論文に登場する定理4の証明を探しています。 LiuとWeinerによる文脈自由言語の交差点の無限階層。 定理4:アン次元アフィンマニホールド寸法である各々がアフィンマニホールドの有限和集合として表現できませんN - 1以下です。nnnn−1n−1n-1 誰かが証拠への言及を知っていますか? 多様体が有限であり、要素に自然順序を定義する場合、格子に関して同様のステートメントはありますか? 定理を理解するための背景: 定義:レッツ有理数の集合とします。部分集合M ⊆ Qは Nであるアフィンマニホールド場合(λ X + (1 - λ )Y )∈ Mときのx ∈ M、Y ∈ M、及びλ ∈ Q。QQ\mathbb{Q}M⊆QnM⊆QnM\subseteq \mathbb{Q}^n(λx+(1−λ)y)∈M(λx+(1−λ)y)∈M(\lambda x+(1-\lambda)y)\in Mx∈Mx∈Mx\in My∈My∈My\in Mλ∈Qλ∈Q\lambda\in\mathbb{Q} 定義:アフィンマニホールドアフィンマニホールドと平行になるようにと言われているMであればM " = M + Aいくつかのため∈ Qのn。M′M′M'MMMM′=M+aM′=M+aM'=M+aa∈Qna∈Qna\in \mathbb{Q}^n 定理:各非空アフィンマニホールドのユニークな部分空間に平行であるK。このKは、で与えられるK = { X - Y :X 、Y ∈ M …

2
完全な格子上の単調関数の一意の固定点(一意の最小/最大の固定点ではない)を保証するための十分な条件
Tarskiの不動点定理は、完全なラティス上の単調演算子の不動点は完全なラティスであると述べています。結果として、完全なラティス上の単調演算子に対して、一意の最大固定点と一意の最小固定点があります。 フィックスポイントは一意にすることもできますが、一般的には多数にすることができます。 私の質問は、単調関数が完全なラティス上で一意の固定点を持つことができるのはどのような条件下ですか?固有のフィックスポイントを保証するための実用的な十分条件はありますか?プロパティを指定する単調演算子がある場合があるため、これを知っておくと便利です。それが本当に指定したい最大の固定点であるか最小の固定点であるかを綴るのは、簡単なことではありません。場合によっては、2つが一致し、上からまたは下から繰り返すと同じ結果が得られることがわかっているので、より単純またはより効率的な方を喜んで選択できます。
弊社のサイトを使用することにより、あなたは弊社のクッキーポリシーおよびプライバシーポリシーを読み、理解したものとみなされます。
Licensed under cc by-sa 3.0 with attribution required.